Tải bản đầy đủ (.pdf) (25 trang)

lovetoan wordpress com CÁC NGUYÊN LÝ

Bạn đang xem bản rút gọn của tài liệu. Xem và tải ngay bản đầy đủ của tài liệu tại đây (273.82 KB, 25 trang )

SỞ GIÁO DỤC VÀ ĐÀO TẠO ĐỒNG NAI
Trường THPT Chuyên Lương Thế Vinh
——————–o0o——————–

NGUYỄN TẤT THU

CHUYÊN ĐỀ LỚP 10 TOÁN

MỘT SỐ NGUN LÝ
TRONG GIẢI TỐN

Biên Hịa - 2018
—————–


2


Mục lục
1 Các phương pháp giải toán
1. Phương pháp quy nạp toán học . . . . . . . . . .
I.
Mở đầu . . . . . . . . . . . . . . . . . . .
II.
Nguyên lí quy nạp . . . . . . . . . . . . .
III.
Một số ví dụ . . . . . . . . . . . . . . . . .
IV.
Bài tập . . . . . . . . . . . . . . . . . . .
2. Nguyên lí Dirichlet . . . . . . . . . . . . . . . . .
I.


Sử dụng nguyên lí Dirichlet . . . . . . . .
II.
Nguyên lí Dirichlet . . . . . . . . . . . . .
1.
Nguyên lí Dirichlet . . . . . . . .
2.
Nguyên lí Dirichle mở rộng . . .
3.
Nguyên lí Dirichle cho tập hợp .
4.
Nguyên lí Dirichle trong hình học
III.
Ví dụ . . . . . . . . . . . . . . . . . . . .
IV.
Bài tập . . . . . . . . . . . . . . . . . . .
3. Nguyên lí cực hạn . . . . . . . . . . . . . . . . . .
I.
Ví dụ . . . . . . . . . . . . . . . . . . . .
II.
Bài tập . . . . . . . . . . . . . . . . . . .
4. Nguyên lí bất biến . . . . . . . . . . . . . . . . .
I.
Ví dụ . . . . . . . . . . . . . . . . . . . .
II.
Bài tập . . . . . . . . . . . . . . . . . . .

3

.
.

.
.
.
.
.
.
.
.
.
.
.
.
.
.
.
.
.
.

.
.
.
.
.
.
.
.
.
.
.

.
.
.
.
.
.
.
.
.

.
.
.
.
.
.
.
.
.
.
.
.
.
.
.
.
.
.
.
.


.
.
.
.
.
.
.
.
.
.
.
.
.
.
.
.
.
.
.
.

.
.
.
.
.
.
.
.

.
.
.
.
.
.
.
.
.
.
.
.

.
.
.
.
.
.
.
.
.
.
.
.
.
.
.
.
.

.
.
.

.
.
.
.
.
.
.
.
.
.
.
.
.
.
.
.
.
.
.
.

.
.
.
.
.

.
.
.
.
.
.
.
.
.
.
.
.
.
.
.

.
.
.
.
.
.
.
.
.
.
.
.
.
.

.
.
.
.
.
.

.
.
.
.
.
.
.
.
.
.
.
.
.
.
.
.
.
.
.
.

.
.

.
.
.
.
.
.
.
.
.
.
.
.
.
.
.
.
.
.

.
.
.
.
.
.
.
.
.
.
.

.
.
.
.
.
.
.
.
.

.
.
.
.
.
.
.
.
.
.
.
.
.
.
.
.
.
.
.
.


.
.
.
.
.
.
.
.
.
.
.
.
.
.
.
.
.
.
.
.

.
.
.
.
.
.
.
.

.
.
.
.
.
.
.
.
.
.
.
.

.
.
.
.
.
.
.
.
.
.
.
.
.
.
.
.
.

.
.
.

.
.
.
.
.
.
.
.
.
.
.
.
.
.
.
.
.
.
.
.

.
.
.
.
.

.
.
.
.
.
.
.
.
.
.
.
.
.
.
.

5
5
5
5
6
9
15
15
15
15
15
15
15
15

18
20
20
21
23
23
24


4

MỤC LỤC


Chương 1
Các phương pháp giải tốn
§1.
I.

Phương pháp quy nạp tốn học

Mở đầu

Quy nạp là phương pháp mạnh được sử dụng nhiều trong Toán học. Trong tổ hợp, phương
pháp quy nạp được sử dụng để giải các bài toán đếm, bài toán chứng inh tồn tại, bài toán cực
trị,. . . Trong bài viết này, tôi xin trao đổi một số áp dụng phương pháp quy nạp vào bài toán
chứng minh tồn tại và cực trị trong tổ hơp.

II.


Nguyên lí quy nạp

Quy nạp toán học là một phương pháp mạnh để chứng minh các phát biểu phụ thuộc vào một
số tự nhiên.
Cho (P (n))n≥0 là một dãy các mệnh đề. Phương pháp quy nạp toán học được sử dụng để chứng
minh P (n) đúng với mọi n ≥ n0 với n0 là một số tự nhiên.
Phương pháp quy nạp toán học (dạng yếu): Giả sử
• P (n0 ) đúng.
• Với mọi k ≥ n0 và P (k) đúng thì P (k + 1) đúng.
Khi đó P (n) đúng với mọi n ≥ n0 .
Phương pháp quy nạp toán học (bước nhảy s): Cho s là số nguyên dương. Giả sử
• P (n0 ) , P (n0 + 1) , . . . , P (n0 + s − 1) đúng.
• Với mọi k ≥ n0 , P (k) đúng kéo theo P (k + s) đúng .
Khi đó P (n) đúng với mọi n ≥ n0 .
Phương pháp quy nạp toán học (Dạng mạnh): Giả sử
• P (n0 ) đúng
• Với mọi k ≥ n0 , P (m) đúng với mọi m mà n0 ≤ m ≤ k kéo theo P (k + 1) đúng.
Khi đó P (n) đúng với mọi n ≥ n0 .
5


6

CHƯƠNG 1. CÁC PHƯƠNG PHÁP GIẢI TỐN

III.

Một số ví dụ

Ví dụ 1. Chứng minh rằng

12 + 22 + 32 + · · · + n2 =

n(n + 1)(2n + 1)
.
6

(1)

k(k + 1)(2k + 1)
6

(1.1).

với mọi số tự nhiên n ≥ 1.
Lời giải.
Với n = 1, ta có (1) đúng, vì hai vế cùng bằng 1.
Giả sử (1) đúng với n = k, k ≥ 1. Tức là
12 + 22 + 32 + · · · + k 2 =
ta cần chứng minh (1) đúng với n = k + 1, tức là
12 + 22 + 32 + · · · + k 2 + (k + 1)2 =

(k + 1)(k + 2)(2k + 3)
6

(2.1).

Thật vậy
V T (2.1) =

k(k + 1)(2k + 1)

+ (k + 1)2 = (k + 1)
6

= (k + 1)

k(2k + 1)
+k+1
6

2k 2 + 7k + 6
(k + 1)(k + 2)(2k + 3)
=
= V P (2.1).
6
6

Từ đó ta có đpcm.
Ví dụ 2. Chứng minh rằng với mọi số tự nhiên n ≥ 2 ta ln có
3n > 3n + 1.

(2)

Lời giải.
Với n = 2 ta có: V T (2) = 32 = 9 > V P (2) = 3.2 + 1 = 7 nên (2) đúng với n = 1..
Giả sử (2) đúng với n = k ≥ 2, tức là: 3k > 3k + 1.
Ta chứng minh (1) đúng với n = k + 1, tức là :
3k+1 ≥ 3(k + 1) + 1 = 3k + 4.

(1.2)
(2.2)


Thật vậy:
3k+1 = 3.3k > 3(3k + 1) = 3k + 4 + (6k − 1) > 3k + 4
nên (2.2) đúng.
Vậy bài tốn được chứng minh.
Ví dụ 3. Cho hàm số f : R → R, n ≥ 2là số nguyên . Chứng minh rằng nếu
f (x) + f (x)
x+y
≥f
∀x, y ≥ 0 (3.1) thì với ∀xi ≥ 0,i = 1, n ta có:
2
2
f (x1 ) + f (x2 ) + . . . + f (xn )
≥f
n

x 1 + x2 + . . . + xn
.
n

(3.2)


1.. PHƯƠNG PHÁP QUY NẠP TỐN HỌC

7

Lời giải.
• Ta chứng minh (3.2) đúng với n = 2k , k ≥ 1.
+) Với k = 1 thì (3.2) đúng (do (3.1)).

+) Giả sử (3.2) đúng với n = 2k , ta chứng minh (3.2) đúng với n = 2k+1 . Thật vậy:
f (x1 ) + . . . + f (x2k ) ≥ 2k f
≥ 2k f

x1 + . . . + x2k
f (x2k +1 ) + . . . + f (x2k+1 )
2k
x2k +1 + . . . + x2k+1
.
2k

Do đó:
x1 + . . . + x2k
x2k +1 + . . . + x2k+1
+ 2k f
k
2
2k
x1 + . . . + x2k + x2k +1 + . . . + x2k+1
.
≥ 2k+1 f
2k+1

f (x1 ) + . . . + f (x2k+1 ) ≥ 2k f

Do vậy (3.2) đúng với mọi n = 2k .
• Giả sử (3.2) đúng với mọi n = k + 1 ≥ 3, tức là
f (x1 ) + f (x2 ) + . . . + f (xk+1 )
≥f
k+1


x1 + x2 + . . . + xk+1
.
k+1

(3.3)

x1 + x2 + . . . + xk
.
k

(3.4)

Ta chứng minh (3.2) đúng với n = k, tức là
f (x1 ) + f (x2 ) + . . . + f (xk )
≥f
k
Thật vậy:
Đặt xk+1 =

x1 + x2 + . . . + xk
x
= , áp dụng (3.3) ta có
k
k

x
x
f (x1 ) + f (x2 ) + . . . + f (xk ) + f
x1 + x 2 + . . . +

k ≥f
k .
k+1
k+1

Hay
f (x1 ) + f (x2 ) + . . . + f (xk )
≥f
k

x1 + x2 + . . . + xk
.
k

Vậy bài tốn được chứng minh.
Ví dụ 4. Chứng minh rằng với mỗi số tự nhiên n ≥ 2, luôn tồn tại một tập hợp S có n
số nguyên dương phân biệt sao cho (a − b)2 là ước của a · b với mọi a, b ∈ S.
Lời giải.
Với n = 2 ta chọn S2 = {0, 1}.
Giả sử tồn tại tập Sn = {a1 , a2 , · · · , an } thỏa yêu cầu bài toán. Gọi x là bội chung nhỏ nhất
của các số khác 0 có dạng (a − b)2 với a, b ∈ Sn . Đặt
Sn+1 = {x + a |a ∈ Sn } ∪ {0} .
Khi đó |Sn+1 | = n + 1 và với a, b ∈ Sn+1 ta có :
• Nếu ab = 0 thì (a − b)2 |ab .


8

CHƯƠNG 1. CÁC PHƯƠNG PHÁP GIẢI TỐN
• Nếu ab = 0, khi đó tồn tại i, j ∈ {1, 2, · · · , n} ; i = j sao cho a = x + ai , b = x + aj . Khi đó

(x + ai ) (x + aj ) = x2 + x (ai + aj ) + ai aj ≡ ai aj ≡ 0 mod(ai − aj )2 ,
hay
(a − b)2 = [(x + ai ) − (x + aj )]2 = (ai − aj )2 |(x + ai ) (x + aj ) = ab.

Vậy bài tốn được chứng minh.
Ví dụ 5. Trên đường trịn người ta điền n (n ≥ 3) số sao cho với mỗi số bất kì thì tổng
của hai số kề bên chia hết cho số đó. Chứng minh rằng tổng các thương khơng vượt q
3n.
Lời giải.
Ta chứng minh bài tốn bằng phương pháp quy nạp.
• n = 3 . Gọi ba số đó a, b, c , khi đó ta có

 a + b = xc
b + c = ya .

c + a = zb
Giả sử a = max {a, b, c} ta có
ya = b + c ≤ 2a ⇒ y ∈ {1, 2} .
+) y = 2, ta có a = b = c nên x + y + z = 6.
+) y = 1, ta có b + c = a. Giả sử b ≥ c, khi đó ta có
zb = c + a = 2c + b ⇒ 2c = (z − 1) b ≥ b
nên c ≤ b ≤ 2c.
Suy ra
xc = a + b = c + 2b ≤ 5c ⇒ x ≤ 5
zb = c + a = 2c + b ≤ 3b ⇒ z ≤ 3.
Do đó x + y + z ≤ 9. Do đó, bài tốn đúng với n = 3.
• Giả sử bài tốn đúng với n ≥ 3, ta chứng minh bài toán cũng đúng với n + 1.
+) Nếu n + 1 số đã cho bằng nhau thì ta có đpcm.
+) Xét trường hợp ngược lại, tồn tại a, b, c sao cho b, c xếp cạnh a và a > max {b, c} và b
nằm giữa a và d, c nằm giữa a và e. Ta có


 b + c = ma
a + d = nb .

a + e = pc
Do b + c < 2a nên m = 1 hay b + c = a. Suy ra
c + d = (n − 1) b
.
b + e = (p − 1) c
Do vậy, khi ta bỏ a khỏi vịng trịn thì n số cịn lại vẫn thỏa yêu cầu đề bài, theo giả thiết
quy nạp thì tổng các thương của n số này không vượt quá 3n, do đó tổng các thương của
n + 1 số không vượt quá 3n + 3 = 3 (n + 1).
Vậy bài toán được chứng minh.


1.. PHƯƠNG PHÁP QUY NẠP TOÁN HỌC

IV.

9

Bài tập

Bài 1. Chứng minh các đẳng thức và bất đẳng thức sau
a) 1.22 + 2.32 + 3.42 + . . . + (n − 1).n2 =
b) 22 + 42 + . . . + (2n)2 =
c)
d)

n(n2 − 1)(3n + 2)

với ∀n ≥ 2.
12

2n(n + 1)(2n + 1)
với ∀n ∈ N∗.
3

1 3 5
2n + 1
1
. . ....
<√
.
2 4 6
2n + 2
3n + 4



1
1
1
n < 1 + √ + √ . . . . + √ ≤ 2 n với ∀n ≥ 1.
n
2
3

e) 1 +

1 1

1
+ + ... + n
< n; (∀n ∈ N∗ , n ≥ 2).
2 3
2 −1

Bài 2. Chứng minh rằng với mọi số nguyên dương n, ta có
a) n(2n2 − 3n + 1) chia hết cho 6.
b) 11n+1 + 122n−1 chia hết cho 133.
c) n5 − n chia hết cho 5
d) 13n − 1chia hết cho 6
e) 16n − 15n − 1chia hết cho 225
f) 4.32n+1 + 32n − 36chia hết cho 64.
Bài 3. Trong mặt mặt phẳng cho n điểm rời nhau (n ≥ 3) tất cả không nằm trên một đường
thẳng. Chứng minh rằng tất cả các đường thẳng nối hai điểm trong các điểm đã cho tạo ra số
đường thẳng khác nhau không nhỏ hơn n.
Bài 4. Tập A = {a1 , a2 , . . . , an } gồmn > 2 số nguyên dương phân biệt được gọi là tập “ tốt”
nếu a2016
luôn chia hết cho tích a1 a2 . . . ai−1 ai+1 . . . an với mọi i = 1, 2016. Tìm số nguyên dương
i
n lớn nhất để tồn tại một tập “tốt” có đúng n phần tử.
Lời giải.
Giả sử A = {a1 , a2 , . . . , an } là tập “tốt” với a1 < a2 < . . . < an . Đặt P1 = a2 a3 . . . an , theo đề bài
ta có P1 |a2016
nên P1 ≤ a2016
. Mặt khác P1 > an−1
nên
1
1
1

n − 1 < 2016 ⇒ n < 2017 ⇒ n ≤ 2016.
Ta chứng min với mọi 3 ≤ n ≤ 2016 luôn tồn tại một tập “tốt” có n phần tử.
Với n = 3 ta chọn A = {6, 18, 36}.
Giả sử A = {a1 , a2 , . . . , ak } là tập “tốt” với k < 2016.
Đặt P = a1 a2 . . . ak và
b0 = P, b1 = a1 P, b2 = a2 P, . . . , bk = ak P.
Khi đó
b1 b2 . . . bk = P k a1 a2 . . . ak P 2016 = b2016
0

b0 b1 . . . bi−1 bi+1 . . . bk = P k a1 ..ai−1 ai+1 . . . ak b2016
.
i
Do đó tập B = {b0 , b1 , . . . , bk } là tập “tốt” có k + 1 phần tử.
Theo ngun lí quy nạp ta có đpcm.


10

CHƯƠNG 1. CÁC PHƯƠNG PHÁP GIẢI TOÁN

Bài 5. Chứng minh rằng với ∀n ≥ 1, ∀x > 0 ta có BĐT:
xn (xn+1 + 1)

xn + 1

x+1
2

2n+1


.

Đẳng thức xảy ra khi nào?
Lời giải.
Gọi (1) là BĐT cần chứng minh.
Với n = 1 ta cần chứng minh:
x(x2 + 1)

x+1

x+1
2

3

⇔ 8x(x2 + 1) ≤ (x + 1)4 .

Hay là:
x4 − 4x3 + 6x2 − 4x + 1 ≥ 0 ⇔ (x − 1)4 ≥ 0 (đúng).
Suy ra đúng với n = 1. Đẳng thức xảy ra khi x = 1.
Giả sử (1) đúng với n = k ≥ 1, tức là:
xk (xk+1 + 1)

xk + 1

x+1
2

2k+1


.

(1.1)

Ta cần chứng minh:
xk+1 (xk+2 + 1)

xk+1 + 1

x+1
2

2k+3

.

(1.2)

Thật vậy, ta có:
x+1
2

2k+3

=

x+1
2


2

x+1
2

2k+1



x+1
2

2

xk (xk+1 + 1)
xk + 1

Nên để chứng minh (1.2) ta chỉ cần chứng minh
2

x+1
2

xk (xk+1 + 1)
xk+1 (xk+2 + 1)

.
xk + 1
xk+1 + 1


Hay
x+1
2

2

2

(xk+1 + 1) ≥ x(xk+2 + 1)(xk + 1).

(1.3)

Khai triển (1.3) , biến đổi và rút gọn ta thu được:
2

x2k+2 (x − 1)2 − 2xk+1 (x − 1)2 + (x − 1)2 ≥ 0 ⇔ (x − 1)2 (xk+1 − 1) ≥ 0.
BĐT này hiển nhiên đúng. Đẳng thức có ⇔ x = 1.
Vậy bài toán được chứng minh.
Bài 6. Một người thợ ln phải lát những căn phịng có kích thước 2n × 2n ơ vng bằng các
mảnh đá lát hình đơ-mi-nơ kích thước 1 × 3 ơ vng và các mảnh đá lát hình thước thợ 3 ơ
vng. Anh ta phải lát kín cả căn phịng chỉ trừ lại một ơ vng duy nhất để trống khơng lát đá
để trang trí đặc biệt. Do giá thành của một viên đá lát hình thước thợ đắt hơn nhiều giá thành
của một viên đá lát hình đơ-mi-nơ, cho nên anh ta muốn sử dụng càng ít viên đá lát hình thước
thợ càng tốt. Anh ta nhận thấy rằng dù vị trí ơ đặc biệt ở đâu đi chăng nữa, chưa lần nào anh
ta phải dùng nhiều hơn n viên gạch lát hình thước thợ để hồn thành cơng việc của mình. Hãy
chứng minh rằng khẳng định của người thợ luôn đúng với mọi số nguyên dương n.


1.. PHƯƠNG PHÁP QUY NẠP TỐN HỌC


Thước thợ

11

Đơ-mi-nơ

Lời giải.
Trước hết ta chứng minh các bổ đề sau:
Bổ đề 1. Mọi hình chữ nhật có kích thước một cạnh chia hết cho 3 thì lát được bởi các viên đá
hình đơ-mi-nơ 1 × 3.
Bổ đề này là hiển nhiên.
Bổ đề 2. Mọi hình thước thợ tạo bởi ba hình vng kích thức 2n × 2n đều lát được bởi một số
viên đá hình đơ-mi-nơ và 1 viên đá hình thước thợ.
Chứng minh:
Kết luận hiển nhiên đúng khi n = 1.
Giả sử kết luận đúng cho n = k − 1 thì ta chia một hình thước thợ tạo bởi 3 hình vng kích
thước 2k × 2k thành các hình chữ nhật có một cạnh chia hết cho 3 và một thước thợ tạo bởi 3
hình vng kích thước 2k−1 × 2k−1 (như trong hình 1).

Hình 1

Hình 2

Dễ thấy các hình chữ nhật thu được đều có đồ dài một cạnh chia hết cho 3 nên chúng được lát
bởi các viên đá đơ-mi-nơ theo Bổ đề 1.
Hình thước thợ gồm 3 ơ vng 2k−1 × 2k−1 theo giả thiết quy nạp lạt được bởi các viên đá
đo-mi-nơ và một viên đá hình thức thợ.
Trở lại bài tốn.
Với n = 1 thì bài toán hiển nhiên đúng.
Giả sử kết luận của bài toán đúng với n = k − 1 ≥ 1. Xét một căn phịng kích thước 2k × 2k ơ

vng. Chia hình vng này thành 4 hình vng bằng nhau thì ô trống sẽ rời vào một trong 4
hình vuông này, chẳng hạn rời vào góc phần tư thứ nhất (hình 2). Theo giả thiết quy nạp thì
cần khơng q k − 1 viên đá lát hình thước thợ để lát hình vng ở góc phần tư thứ nhất. Để
lát hình thước thợ gồm 3 hình vng kích thức 2k−1 × 2k−1 , thi theo Bổ đề 2 ta cần một viên
đá hình thước thợ. Do đó, ta cần khơng qua (k − 1) + 1 = k viên đá hình thước thợ để lát căn
phịng kích thước 2k × 2k .
Vậy bài toán được chứng minh.
Bài 7. Một tập hợp con A có ít nhất ba phần tử của tập các số nguyên dương được gọi là tập
“thô” nếu với 3 phần tử phân biệt a, b, c của A thì a khơng là bội của (b, c), với (x, y) là kí
hiệu ước chung lớn nhất của hai số nguyên x và y. Chứng minh rằng nếu A là tập “thơ” có n
phần tử và n ≥ 4 thì max A ≥ 4 (n2 − n − 1) .
Lời giải.
Bổ đề 1. Từ 2n − 2 số tự nhiên đầu tiên ta chọn ra n số, khi đó ln có hai trong n số đã chọn
là bội của nhau.


12

CHƯƠNG 1. CÁC PHƯƠNG PHÁP GIẢI TOÁN

Thật vậy:
Xét n số 2ki mi với ki là các số tự nhiên và mi là các số nguyên dương lẻ.
Vì trong tập {1, 2, . . . , 2n − 2} có đúng n − 1 số lẻ, suy ra trong n số {m1 , m2 , . . . , mn } có hai số
.
bằng nhau, chẳng hạn mt = ml (t > l). Khi đó 2kt mt ..2kl ml (đpcm).
Nhận xét: Từ bổ đề trên ta thấy:
Nếu lấy n số mà trong đó khơng có hai số nào là bội của nhau thì max của n số đó khơng nhỏ
hơn 2n − 1.
Bổ đề 2. Nếu A = {a, b, c} là tập “thơ” thì max {a, b, c} ≥ 15.
Thật vậy:

Vì {a, b, c} là tập thơ nên (a, b) , (b, c), (c, a) là ba số mà không có hai số nào là bội của nhau.
Theo bổ đề 1, ta có
max {(a, b) , (b, c) , (c, a)} ≥ 5.
Hơn nữa 1 <

b
a
=
nên
(a, b)
(a, b)
max

a
b
,
(a, b) (a, b)

≥ 3.

Từ đó, suy ra
max {a, b, c} ≥ 3 max {(a, b) , (b, c) , (c, a)} ≥ 15.
Trở lại bài toán.
Xét n = 4, đặt A = {a, b, c, d}. Gọi x1 , x2 , · · · , x6 là UCLN của hai trong 4 số thuộc tập A và
x = max {x1 , x2 , · · · , x6 }. Ta có x ≥ 11.
Dễ thấy x ∈
/ A.
+) Nếu kx ∈ A với k ≥ 4 thì max A ≥ 4.11 = 44.
+) Nếu 2x ∈ A, 3x ∈ A. Xét y ∈ A\ {2x, 3x}. Nếu (y, 3) = 1 thì (y, 3x) = (y, x) |2x (vơ lí).
Do đó 3 |y , hay A = {2x, 3x, 3y, 3z}. Do A là tập thô nên B = {x, y, z} cũng là tập thô,

suy ra
max A ≥ 3 max {x, y, z} ≥ 3.15 = 45 > 44.
Suy ra bài toán đúng với n = 4.
Giả sử bài toán đúng đến n − 1. Ta chứng minh bài toán đúng đến n.
Xét A là tập thơ có n phần tử, khi đó có Cn2 UCLN của hai phần tử bất kì thuộc A.
Đặt X = {x1 , x2 , · · · , xk } , k = Cn2 là tập các UCLN và x = max X.
Theo chứng minh tương tự như trường hợp n = 4 ta chỉ xét 2x, 3x ∈ A. Khi đó
A = {2x, 3x, 3a1 , 3a2 , · · · , 3an−2 }
Và tập Y = {x, a1 , a2 , · · · , an−2 } là tập thô, nên
max Y ≥ 4 (n − 1)2 − (n − 1) − 1 .
Suy ra
max A ≥ 12 n2 − 3n + 1 ≥ 4 n2 − n + 1 ∀n ≥ 4.
Vậy bài toán được chứng minh.
Bài 8. Cho n (n ≥ 2) số thực bất kì có tổng bằng 0. Hỏi trong n số đó có ít nhất bao nhiêu cặp
có tổng khơng âm.
Lời giải.
Ta gọi một cặp số thực có tổng không âm là một cặp "đẹp" và f (n) là số cặp đẹp ít nhất trong
n số thưc có tổng bằng 0. Ta có các nhận xét sau:


1.. PHƯƠNG PHÁP QUY NẠP TOÁN HỌC

13

Nhận xét 1. Nếu có n số thực có tổng bằng k > 0 thì số cặp đẹp khơng nhỏ hơn f (n).
Nhận xét 2. Xét n số gồm n − 1 số −1 và số n − 1, khi đó ta thấy có n − 1 cặp đẹp, do đó
f (n) ≤ n − 1.
Ta thấy f (2) = 1, f (3) = 1, f (4) = 3, f (5) = 3, f (6) = 5. Ta chứng minh f (n) = n − 1, ∀n ≥ 6
bằng cách chứng minh
f (n + 1) ≥ f (n) + 1 ∀n ≥ 6

.(1)
Thật vậy, ta xét n + 1 số thực có tổng bằng 0.
• Nếu trong n + 1 số có một số bằng 0 thì ta loại số đó ra, khi đó n số cịn lại có tổng bằng 0
nên số cặp đẹp trong n số này là f (n). Vì trong n số này có ít nhất một số khơng a không
âm nên 0 và a tạo thêm một cặp đẹp, dẫn đến (1) đúng.
• Xét n + 1 số khác 0. Gọi A, B lần lượt là tập các số âm và tập các số dương trong n + 1
số đã cho. Ta có |A| + |B| = n + 1. Xét a = min A, a < 0, ta loại a thì n số cịn lại có tổng
dương nên số cặp đẹp trong n số này không nhỏ hơn f (n).
+) Nếu max B ≥ min A thì tồn tại b ∈ B để a + b ≥ 0, khi đó số cặp đẹp sẽ thêm 1, nên
(1) đúng.
2
+) Nếu max B < min A thì |A| < |B| và f (n + 1) = C|B|
. Khi đó, nếu n + 1 = 2k + 1 thì
|B| ≥ k + 1 nên
2
f (n + 1) ≥ Ck+1
=

k(k + 1)
≥ 2k ≥ f (n) + 1 ∀k ≥ 3 (n ≥ 6).
2

Nếu n = 2k thì |B| ≥ k + 1 và chứng minh tương tự như trên.
Bài 9. Cho n ≥ 2 con chim đậu trên đường tròn (O), mỗi điểm có nhiều nhất một con chim
0

đậu. Hai con chim đậu tại điểm Pi , Pj được gọi là trơng thấy nhau nếu P
i OPj ≤ 120 . Tìm theo
n số nhỏ nhất các cặp con chim trông thấy nhau.
Lời giải.

Gọi f (n) là số nhỏ nhất các cặp con chim trông thấy nhau.
(n − 1)2
Ta chứng minh f (n) =
.
4
Ta có f (2) = 0, f (3) = 1 nên bài toán đúng với n = 2, n = 3.
(n − 1)2
n2
Giả sử f (n) =
, n ≥ 2. Ta chứng minh f (n + 1) =
.
4
4
Theo nguyên tắc cực hạn thì trong n + 1 con chim có một con chim trơng thấy nhiều con chim
nhất, giả sử đó là con chim đậu ở điểm A (ta gọi tắt là A) và m là số con chim mà A nhìn thấy .
Trên đường trịn lấy hai điểm X, Y sao cho
÷ = XOY
÷ =Y
÷
AOX
OA = 1200 .

Vì A trông thấy m con chim nên M con chim này nằm trên cung XAY tính cả hai điểm mút.
Do đó, trên cung nhỏ XY có n + 1 − (m + 1) = n − m con chim, và mỗi con chim này trông thấy
n − m − 1 con chim cịn lại.
Suy ra số cặp chim trơng thấy, trong đó có A là m và do tính chất của A nên ta có
m≥n−m−1⇒m≥

n−1
n


.
2
2

Số nhỏ nhất các cặp chim thấy nhau của n con chim còn lại (trừ A) là f (n) và rõ ràng các cặp
chim trông thấy nhau này khác với m cặp chim trông thấy nhau ở trên. Do đó, ta có
f (n + 1) = f (n) + m ≥

(n − 1)2
n
+
.
4
2


14

CHƯƠNG 1. CÁC PHƯƠNG PHÁP GIẢI TOÁN

Ta chứng minh
(n − 1)2
n
n2
+
=
.
4
2

4

(1)

+) n = 2k thì
V T (1) = k (k − 1) +

1
n2
+ [k] = k (k − 1) + k = k 2 =
= V P (1).
4
4

+) n = 2k + 1 thì
V T (1) = k 2 + k +

1
n2
= k2 + k =
= V P (1).
2
4

(n − 1)2
Do đó, ta chứng minh được f (n) ≥
. Ta chỉ ra trường hợp có dấu “=”.
4
n
Trên đường trịn lấy hai điểm X, Y sao cho cung sđXY = 300 . Cho

con chim đậu trên cung
2
n+1
n
=
con chim còn lại đầu trên cung nhỏ X Y đối xứng với cung XY
nhỏ XY và n −
2
2
qua tâm O. Khi đó, các con chim nằm trên hai cung XY và X Y trông thấy nhau , đồng thời
hai con chim bất kì nằm trên hai cung XY và X Y đều không trông thấy nhau. Do đó, số cặp
chim trơng thấy nhau là
n
2

n+1
n
−1
2
2
+
2

n+1
−1
2
2

=


(n − 1)2
.
4


2.. NGUN LÍ DIRICHLET

§2.
I.

15

Ngun lí Dirichlet

Sử dụng ngun lí Dirichlet

Ngun lí Dirichle (hay là nguyên lí chuồng thỏ) được phát biểu hết sức đơn giản nhưng lại có
nhiều ứng dụng trong tốn học và đặc biệt ngun lí Dirichle là một cơng cụ mạnh để chứng
minh bài tốn tồn tại. Sau đây, chúng ta đi xét một số ứng dụng của ngun lí Dirichle cho bài
tốn tồn tại.

II.
1.

Ngun lí Dirichlet
Ngun lí Dirichlet

Nếu nhốt n + 1 con thỏ vào n chuồng thì có một chuồng chứa ít nhất 2 con thỏ.
2.


Nguyên lí Dirichle mở rộng

Nếu nhốt n con thỏ vào m chuồng (m ≤ n) thì có một chuồng chứa ít nhất
3.

n+m−1
con.
m

Nguyên lí Dirichle cho tập hợp
m

|Si | > k. |S|. Khi đó

Cho S là tập hợp hữu hạn. S1 , S2 , . . . , Sm là các tập con của S sao cho
i=1

tồn tại một phần tử x ∈ S sao cho x chứa ít nhất trong k + 1 tập của họ {S1 , S2 , . . . , Sm }.
4.

Ngun lí Dirichle trong hình học

Cho một hình phẳng (H) và (Hi ), i = 1, n là các hình phẳng nằm trong (H). Kí hiệu S, Si là
n

diện tích của các hình phẳng (H) và (Hi ). Khi đó, nếu S <

Si thì tồn tại hai hình phẳng
i=1


(Hi ), (Hj ) có giao khác rỗng với i, j ∈ {1, 2, . . . , n}.
Để sử dụng nguyên lí Dirichle, ta cần tạo ra số chuồng và số thỏ.

III.

Ví dụ

Ví dụ 1. Chứng minh rằng trong n + 1 số bất kì thuộc tập hợp {1, 2, 3, . . . , 2n} luôn chọn
được hai số mà số này là bội của số kia.
Lời giải.
Viết n + 1 số đã cho dưới dạng:
a1 = 2k1 b1 , a2 = 2k2 b2 , . . . , an+1 = 2kn+1 bn+1 .
Trong đó b1 , b2 , . . . , bn+1 là các số lẻ. Ta có: 1 ≤ b1 , b2 , . . . , bn+1 ≤ 2n − 1. Mặt khác trong khoảng
từ 1 đến 2n − 1 có đúng n số lẻ nên tồn tại hai số m < n sao cho bn = bm . Khi đó, trong hai số
an và am có một số là bội của số kia.
Ví dụ 2. Chọn bất kì n + 1 số trong 2n số tự nhiên từ 1 đến 2n (n ≥ 2). Chứng minh
rằng trong các số được chọn có ít nhất 1 số bằng tổng của 2 số được chọn (kể cả các trường
hợp 2 số hạng của tổng bằng nhau ).
Lời giải.


16

CHƯƠNG 1. CÁC PHƯƠNG PHÁP GIẢI TOÁN

Giả sử a1 < a2 < . . . < an < an+1 là n + 1 số được chọn.
Xét n số: an+1 − a1 = b1 , an+1 − a2 = b2 , . . ., an+1 − an = bn .
Trong tập 2n + 1 số đó là a1 , a2 , . . . , an+1 , b1 , b2 , . . . , bn tồn tại 2 số bằng nhau, hai số ấy không
thể cùng thuộc dãy a1 , a2 , . . . , an+1 cũng không thể cùng thuộc dãy b1 , b2 , . . . , bn . Ta có:
an+1 − a1 = ai , suy ra an+1 = a1 + ai .


Ví dụ 3. Trong mặt phẳng cho 2n + 1 điểm sao cho với 3 điểm bất kì ln có 2 điểm mà
khoảng cách giữa chúng nhỏ hơn 1. Chứng minh rằng tồn tại một đường trịn có bán kính
bằng 1 chứa ít nhất n + 1 điểm trong 2n + 1 điểm đã cho.
Lời giải.
Xét A là một trong 2n + 1 điểm. Xét đường tròn (S) = (A, 1).
Nếu (S) chứa 2n điểm cịn lại thì ta có điều phải chứng minh.
Nếu B ∈
/ (S), ta xét đường tròn (S ) = (B, 1).
Khi đó với điểm C bất kì khác A và B, ta có:
AC < 1
C ∈ (S)

.
BC < 1
C ∈ (S )
Do đó trong 2n − 1 điểm còn lại ( khác A và B) hoặc thuộc (S) hoặc thuộc (S ) nên trong hai
đường trịn đó chứa ít nhất n điểm. Hay đường trịn đó chứa ít nhất n + 1 điểm trong 2n + 1
điểm đã cho.
Ví dụ 4. Cho đa giác lồi 2018 cạnh có các tọa độ nguyên. Chứng minh rằng trong đa giác
có ít nhất 403 điểm có tọa độ nguyên.
Lời giải.
Xét 5 đỉnh liên tiếp của đa giác A, B, C, D, E. Vì
(A + B) + (B + C) + (C + D) + (D + E) + (E + A) = 2(A + B + C + D + E) = 6 · 180◦ .
Suy ra trong 5 đỉnh A, B, C, D, E luôn tồn tại hai đỉnh chung cạnh và tổng của hai góc đó
lớn hơn 180◦ . Giả sử hai góc đó là A + B > 180◦ . Mặt khác:
A + B + C1 + D1 = 360◦ ⇒

A + E1 > 180◦
.

B + C1 > 180◦

Giả sử B + C1 > 180◦ . Dựng hình bình hành ABCF , suy ra F nằm trong tứ giác ABCE.
Vì ABCF là hình bình hành và A, B, C có tọa độ nguyên nên dẫn tới F cũng có tọa độ ngun.
Từ đó ta có đpcm.
Ví dụ 5. Trên bàn cờ 10 × 10 người ta viết các số từ 1 đến 100. Mỗi hàng chọn ra số lớn
thứ ba. Chứng minh rằng tồn tại một hàng có tổng các số trong hàng đó nhỏ hơn tổng các
chữ số lớn thứ ba được chọn.
Lời giải.
Kí hiệu các số lớn thứ ba là a9 < a8 < . . . < a0 . Khi đó số phần tử lớn hơn a0 nhiều nhất là 20
(nhiều nhất là 2 phần tử ở mỗi hàng). Suy ra a0 ≥ 80 (1).
Tương tự a1 ≥ 78 (2).
Mặt khác a8 ≥ a9 + 1, a7 ≥ a9 + 1, . . . , a2 ≥ a9 + 7. Kết hợp với (1) và (2) suy ra:
a0 + a1 + . . . + a9 ≥ 80 + 78 + (a9 + 1) + . . . + (a9 + 7) = 8a9 + 180.


2.. NGUYÊN LÍ DIRICHLET

17

Xét hàng chứa a9 . Tổng các số của dòng chứa a9 là
S(a9 ) ≤ 100 + 99 + a9 + a9 − 1 + . . . + a9 − 7
= 8a9 + 171 < 8a9 + 180 ≤ a0 + a1 + . . . + a9 (đpcm).

Ví dụ 6. Trong một cuộc thi Tốn có 65 học sinh tham gia đến từ hai trường. Mỗi học
sinh thi một trong 4 mơn Tốn, Lí, Hố, Anh Văn. Biết rằng trong 5 học sinh thi cùng
một mơn thì có hai học sinh cùng tuổi. Chứng minh rằng trong 65 học sinh có ít nhất 3
học sinh đến từ một trường, thi cùng một môn và bằng tuổi nhau.
Lời giải.
Giả sử khơng có 3 học sinh nào thoả u cầu bài tốn.

Vì có 65 học sinh đến từ hai trường nên có ít nhất 33 học sinh đến từ một trường. Xét 33 học
này thì có ít nhất 9 học sinh thi cùng một môn.
Ta xét 9 học sinh này:
Lấy 5 học sinh bất kì trong 9 học sinh trên. Khi đó sẽ có hai học sinh cùng tuổi, ta giả sử đó là
hai học sinh A1 , B1 . Ta loại hai học sinh này còn lại 7 trong học sinh và trong 7 học sinh nay ta
tìm được hai học sinh cùng tuổi A2 , B2 . Sau khi loại hai học sinh này ta còn lại 5 học sinh và
tiếp tục chọn được 2 học sinh A3 , B3 .
Xét 3 học sinh A1 , A2 , A3 ta có tuổi của ba học sinh này đôi một khác nhau.
Xét 5 học sinh gồm ba học sinh A1 , A2 , A3 với 2 trong ba học sinh cịn lại, khi đó hai học sinh
cịn lại ta kí hiệu A4 , B4 cùng tuổi nhau.
Xét 5 học sinh gồm 4 học sinh A1 , A2 , A3 , A4 và học sinh còn lại (ta kí hiệu là A5 ). Khi đó A5 sẽ
cùng tuổi với 1 trong 4 học sinh A1 , A2 , A3 , A4 . Chẳng hạn A5 , A1 cùng tuổi. Khi đó A1 , B1 , A5
thoả yêu cầu bài toán . Điều này mâu thuẫn với điều giả sử ở trên.
Vậy bài tốn được chứng minh.
Ví dụ 7. Mỗi đỉnh của một cửu giác đều (đa giác đều 9 cạnh) được tô một trong hai màu
xanh hoặc đỏ. Chứng minh rằng tồn tại hai tam giác có đỉnh là đỉnh của cửu giác, đồng
dạng với nhau và các đỉnh được tô cùng một màu.
Lời giải.
A6
A5

A7

A4

A8
O

A3


A9

A2

A1

Chúng ta gọi một tam giác đỏ (xanh) nếu tất cả các đỉnh của tam giác được tơ màu đỏ
(xanh).
Vì 9 đỉnh của cửu giác được tô bởi hai màu nên theo nguyên lí Drichle, có ít nhất 5 đỉnh được


18

CHƯƠNG 1. CÁC PHƯƠNG PHÁP GIẢI TỐN

tơ một màu. Ta giả sử 5 đỉnh được tô màu đỏ. Suy ra có ít nhất C53 = 10 tam giác đỏ, kí hiệu T
là tập gồm 10 tam giác đỏ này. Ta chứng minh trong 10 tam giác đỏ này, có hai tam giác đồng
dạng với nhau.
Đặt cửu giác đó là A1 A2 ...A9 và đường tròn (O) là đường tròn ngoại tiếp cửu giác. Khi đó cửu
giác sẽ chia đường trong (O) thàng 9 cung nhỏ bằng nhau. Ta gọi mỗi cung nhỏ là một “lá”.
Xét tam giác Ai Aj Ak với Ai Aj ≤ Aj Ak ≤ Ak Ai . Gọi aij là số “lá” nằm trên cung Ai Aj không
chứa Ak ; ajk , aki được định nghĩa tương tự. Ta thấy mỗi tam giác Ai Aj Ak tương ứng với một bộ
(aij , ajk , aki ) thỏa 1 ≤ aij ≤ ajk ≤ aki ≤ 7 và aij + ajk + aki = 9. Ví dụ tam giác A3 A5 A9 tương
ứng với bộ (2; 3; 4). Chia T thành các tập con Ti mà mỗi tập con Ti chứa các tam giác đồng dạng
a+b+c=9
thuộc T . Như vậy mỗi Ti tương ứng với một bộ nghiệm của phương trình
1≤a≤b≤c≤7
(*) và ngược lại. Phương trình (*) có 7 bộ nghiệm: (1; 1; 7) , (1; 2; 6), (1; 3; 5), (1; 4; 4), (2; 2; 5),
(2; 3; 4) , (3; 3; 3) Do đó ta có 7 tập Ti , mà trong T có 10 tam giác nên theo ngun lí Dirchle,
trong các tập Ti có ít nhất một tập chứa ít nhất hai tam giác. Bài tốn được giải quyết.


IV.

Bài tập

Bài 1. Trong hình vng có cạnh bằng 1 đặt 51 điểm bất kì phân biệt. Chứng minh rằng có ít
1
nhất ba trong số 51 điểm đó nằm trong một hình trịn bán kính .
7
Bài 2. Trên tờ giấy kẻ caro lấy 101 ô vuông bất kì. Chứng minh rằng trong 101 ơ vng đó có
26 ô vuông không chung cạnh hoặc chung đỉnh.
Lời giải.
Tô màu xen kẽ các ô bởi 4 màu. Hàng thứ nhất tô Xanh, Đỏ xen kẽ, hàng thứ hai tô Trắng, Đen
xen kẽ. Khi đó có một màu tơ ít nhất 26 ô.
Bài 3. Xét 100 số nguyên dương a1 , a2 , . . . , a100 ; ai ≤ 100 với i = 1, 2, . . . , 100 và
a1 + a2 + . . . + a100 = 200.
Chứng minh rằng trong 100 số đó ln tồn tại một vài số có tổng bằng 100.
Lời giải.
• Nếu ai = aj với mọi i, j thì ta có điều phải chứng minh.
• Nếu tồn tại hai số khác nhau, chẳng hạn a1 = a2 . Xét các số a1 , a2 , a1 + a2 , a1 + a2 +
a3 , . . . , a1 + a2 + . . . + a99 .
Nếu trong các số trên, có một số chia hết cho 100 thì ta có đpcm.
Nếu trong các số trên, khơng có số nào chia hết cho 100 thì trong các số trên có hai số có
cùng số dư khi chia cho 100. Hiệu hai số đó là tổng cần tìm.
Bài 4. Cho 69 số nguyên dương phân biệt không vượt quá 100. Chứng minh rằng có thể chọn
được 4 số a, b, c, d sao cho a < b < c và a + b + c = d.
Lời giải.
Giả sử các số là 1 ≤ a1 < a2 < . . . < a69 ≤ 100. Khi đó a1 ≤ 32. Xét hai dãy sau:
1 < a1 + a3 < a1 + a4 < ... < a1 + a69 ≤ 132 (1)
.

1 < a3 − a2 < a4 − a2 < ... < a69 − a2 ≤ 132 (2)
(1) và (2) có 134 số hạng trong đoạn [1; 132] suy ra có 2 số bằng nhau thuộc về hai dãy . Ta có
đpcm.


2.. NGUYÊN LÍ DIRICHLET

19

Bài 5. Xét tập hợp S = {(x; y) |x, y ∈ Z, 0 ≤ x, y ≤ 3 }. Chọn ra 9 phần tử thuộc S. Chứng
minh rằng trong 9 phần tử được chọn, tìm được 4 phần tử (x1 ; y1 ) , (x2 ; y2 ) , (x3 ; y3 ) , (x4 ; y4 )
sao cho x1 + x2 + x3 + x4 chia hết cho 4 và y1 + y2 + y3 + y4 chia hết cho 4.
Lời giải.
Ta chia các phần tử của S vào 4 nhóm theo tính chẵn lẻ của các tọa độ: (C; C) , (C; L) , (L; C)
và (L, L). Giả sử trong một nhóm có k phần tử là (x1 , y1 ) , ..., (xk , yk ) thì ta lập các phần tử
x1 + xk y 1 + y k
x1 + x2 y 1 + y 2
,
, ...,
,
. Ta được k − 1 phần tử khác nhau thuộc S.
mới là
2
2
2
2
Từ 4 nhóm ta sẽ có được 9 − 4 = 5 phần tử. Trong 5 phần tử này, có 2 phần tử có tọa độ có
cùng tính chẵn lẻ. Từ đẳng thức
x1 + x2 x3 + x4
+

x1 + x2 + x3 + x4
2
2
=
4
2
ta suy ra điều phải chứng minh.
Bài 6. Cho tập S = {1, 2, 3, . . . , 99} và A là một tập con bất kì của S mà |A| = 835. Chứng
minh rằng luôn tồn tại 4 phần tử a, b, c, d thuộc A sao cho: a + 2b + 3c = d.
Lời giải.
Đặt A = {a1 , a2 , . . . , a835 } với a1 < a2 < . . . < a835 .
Xét hiệu
d = a835 − 3a1 = 3(a835 − a1 ) − 2a835 ≥ 3 · 834 − 2 · 999 = 504.
Do đó 166 cặp số sau là phân biệt (d − 2; 1) , (d − 4; 2), . . . , (d − 2.165; 165), (d − 2.166; 166).
Vì có 164 phần tử S khơng thuộc tập A, nên trong các cặp trên tồn tại ít nhất một cặp (x; y)
với y = a1 mà x, y ∈ A, giả sử cặp đó là (d − 2k; k) với k ∈ {1, 2, . . . , 166} . Khi đó ta có ngay:
x + 2y + 3a1 = a835 , suy ra đpcm.
Bài 7. Trong mặt phẳng Oxy, cho 19 điểm có toạ độ nguyên, trong đó khơng có 3 điểm nào
thẳng hàng. Chứng minh rằng có ít nhất 3 điểm trong 19 điểm đã cho tạo thành một tam giác
có trọng tâm là các điểm có toạ độ là số nguyên.
Bài
√ 8. Cho ngũ giác lồi ABCDE có độ dài mỗi cạnh và độ dài các đường chéo không vượt quá
3 . Lấy 2011 điểm phân biệt tùy ý nằm trong ngũ giác đó. Chứng minh rằng tồn tại một hình
trịn đơn vị có tâm nằm trên cạnh của ngũ giác đã cho chứa ít nhất 403 điểm trong số các điểm
đã lấy.


20

CHƯƠNG 1. CÁC PHƯƠNG PHÁP GIẢI TỐN


§3.

Ngun lí cực hạn

Một tập hợp hữu hạn các số thực ln có phần tử lớn nhất và phần tử nhỏ nhất. Một tập con
bất kỳ của N ln có phần tử nhỏ nhất. Nguyên lý đơn giản này trong nhiều trường hợp rất có
ích cho việc chứng minh. Hãy xét trường hợp biên! Đó là khẩu quyết của ngun lý này.

I.

Ví dụ
Ví dụ 1. Có 3 nhóm học sinh, mỗi nhóm có 100 em. Biết mỗi học sinh ở nhóm này quen
với ít nhất 101 em học sinh ở hai nhóm cịn lại. Chứng minh rằng ta có thể chọn ra mỗi
nhóm một học sinh sao cho ba học sinh đó đơi một quan nhau.

Lời giải.
Gọi A là học sinh có nhiều bạn nhất ở một trường khác. Gọi số bạn nhiều nhất này là k. Giả sử
A ở trường thứ nhất và tập những bạn quen A là M = {B1 , B2 , . . . , Bk } ở trường thứ 2. Cũng
theo giả thiết, có ít nhất 1 học sinh C ở trường thứ 3 quen với A. Vì C quen không quá k học
sinh ở trường thứ nhất nên theo giả thiết C quen với ít nhất n + 1 − k học sinh của trường thứ
hai, đặt N = {D1 , D2 , . . . , Dm } là những người quen C ở trường thứ hai thì m ≥ n + 1 − k.
Vì M , N đều thuộc tập hợp gồm n học sinh và |M | + |N | ≥ k + n + 1 − k = n + 1 nên ta có
M ∪ N = ∅. Chọn B nào đó thuộc M ∪ N thì ta có A, B, C đơi một quen nhau.
Ví dụ 2. Chứng minh rằng khơng tồn tại n > 1 lẻ để 15n + 1 chia hết cho n.
Lời giải.
Giả sử tồn tại một số nguyên lẻ n > 1 sao cho 15n + 1 chia hết cho n. Gọi p là ước số nguyên tố
nhỏ nhất của n, khi đó p lẻ. Giả sử k là số nguyên dương nhỏ nhất sao cho 15k − 1 chia hết cho
p (số k được gọi là bậc của 15 theo modulo p).
Vì 152n − 1 = (15n − 1)(15n + 1) chia hết cho p. Mặt khác, theo định lý nhỏ Fermat thì 15p−1 − 1

chia hết cho p. Theo định nghĩa của k, suy ra k là ước số của các số p − 1 và 2n. Suy ra
k| (p − 1, 2n). Do p là ước số nguyên tố nhỏ nhất của n nên (n, p − 1) = 1. Suy ra (p − 1, 2n) = 2.
Vậy k| 2. Từ đó k = 1 hoặc k = 2. Cả hai trường hợp này đều dẫn tới p = 7. Nhưng điều này
mâu thuẫn vì 15n + 1 ln đồng dư 2 mod 7.
Ví dụ 3. Cho hai số nguyên dương a, b nguyên tố cùng nhau. Chứng minh rằng tồn tại
hai số nguyên x, y sao cho ax + by = 1.
Lời giải.
Đặt A = {ax + by > 0 |x, y ∈ Z}. Vì a = 1.a + 0.b, b = 0.a + 1.b nên a, b ∈ A, do đó A = ∅. Suy
ra trong tập A luôn tồn tại phần tử nhỏ nhất. Gọi d là phần tử nhỏ nhất của tập A, ta chứng
minh d = 1. Đặt a = dq + r, 0 ≤ r < d. Vì d ∈ A và d ≤ a nên q > 0 tồn tại x, y ∈ Z sao cho
d = xa + yb.
Suy ra
qd = axq + byq ⇔ a − r = axq + byq ⇔ r = (1 − xq) a − yqb = x a + y b,
.
với x , y ∈ Z. Do tính nhỏ nhất của d nên ta có r = 0 hay a..d. Chứng minh tương tự, ta cũng
.
có b..d. Mà (a, b) = 1 nên ta có d = 1. Bài tốn được chứng minh.
Ví dụ 4. Trong mặt phẳng cho 2015 đường thẳng phân biệt sao cho ba đường thẳng bất
kì ln đồng quy tại một điểm. Chứng minh rằng 2015 đường thẳng trên cùng đi qua một
điểm.


3.. NGUYÊN LÍ CỰC HẠN

21

Lời giải.
Giả sử 2015 đường thẳng đã cho không đồng quy tại một
A
K

điểm. Xét tất cả các khoảng cách khác 0 từ các giao điểm
của 2015 đường thẳng đã cho đến các đường thẳng đó. Vì
E
các khoảng cách này là hữu hạn nên tồn tại một khoảng
cách h nhỏ nhất. Giả sử h = d (A, ∆) = AH với A là một
trong các giao điểm, ∆ là một trong 2015 đường thẳng
B
H
C
D
đã cho và H là chân đường vng góc hạ từ A xuống ∆.
Vì qua A có ít nhất ba đường thẳng đi qua. Giả sử ba đường thẳng đó lần lượt cắt đường thẳng
∆ tại các điểm B, C, D. Trong ba điểm B, C, D có ít nhất 2 điểm nằm cùng phía so với điểm H,
ta giả sử đó là C, D và HC < HD. Gọi E, K là hình chiếu vng góc của C và H lên đường
thẳng AD. Ta có CE < HK < AH , suy ra vơ lí vì AH là khoảng cách nhỏ nhất. Vậy bài toán
được chứng.

II.

Bài tập

Bài 1. Cho n điểm xanh và n điểm đỏ trên mặt phẳng, trong đó khơng có 3 điểm nào thẳng
hàng. Chứng minh rằng ta có thể nối 2n điểm này bằng n đoạn thẳng có đầu mút khác màu sao
cho chúng đôi một không giao nhau.
Bài 2. Trên mặt phẳng cho 2 × 2011 điểm, trong đó khơng có bất kỳ 3 điểm nào thẳng hàng.
Người ta tô 2011 điểm bẳng màu đỏ và tô 2011 điểm còn lại bằng màu xanh. Chứng minh rằng:
bao giờ cũng tồn tại một cách nối tất cả các điểm màu đỏ với tất cả các điểm màu xanh bởi 2011
đoạn thẳng khơng có điểm nào chung.
Lời giải.
Ta nhận thấy rằng luôn tồn tại cách nối 2011 cặp điểm với nhau bằng 2011 đoạn thẳng và vì có

2011 cặp điểm nên số cách nối là hữu hạn và nếu dùng tổ hợp thì ta có thể tính được con số
chính xác các cách nối. Và hiển nhiên là trong hữu hạn cách nối đó ta ln tìm ra được một cách
nối có tổng độ dài các đoạn thẳng là ngắn nhất. Ta chứng minh cách nối đó là cách mà chúng
ta cần tìm.
Thật vậy: Giả sử ngược lại ta có hai đoạn thẳng AX và BY mà cắt nhau tại điểm O ( giả sử A
và B tô màu đỏ, cịn X và Y tơ màu xanh).
Khi đó, nếu ta thay đoạn thẳng AX và BY bằng hai đoạn AY và BX, các đoạn cịn lại giữ
ngun thì ta có cách nối này có tính chất:
AY + BX < (AO + OY ) + (BO + OX)
= (AO + OX) + (BO + OY )
⇒ AY + BX < AX + BY.
Như vậy, việc thay hai đoạn thẳng AX và BY bằng hai đoạn thẳng AY và BX , ta nhận được
một cách nối mới có tổng độ dài đoạn thẳng là nhỏ hơn. Vơ lý, vì trái với giả thiết là đã chọn
cahcs nối có tổng các độ dài là bé nhất.
Điều vơ lí đó chứng tỏ cách nối có tổng độ dài các đoạn thẳng là ngắn nhất là khơng có điểm
chung.
Bài 3. Một nước có 80 sân bay, mà khoảng cách giữa hai sân bay nào cũng khác nhau. Mỗi máy
bay cất cánh từ một sân bay và bay đến sân bay nào gần nhất. Chứng minh rằng: trên bất kỳ
sân bay nào cũng khơng thể có quá 5 máy bay đến.
Lời giải.
Từ giả thiết suy ra nếu các máy bay tư các sân bay M và N đến sân bay O thì khoảng cách

M N là lớn nhất trong các cạnh của tam giác M ON , do đó M
ON > 60◦ .
Giả sử rằng các máy bay bay từ các sân bay M1 , M2 , M3 , M4 , . . . ,Mn đến sân bay O thì một


22

CHƯƠNG 1. CÁC PHƯƠNG PHÁP GIẢI TOÁN


360◦
Ÿ
(i, j, n = 1, 2, 3, 4, 5, . . . , 80 ) vì tổng các góc đã cho
trong các góc M
OM
khơng
lớn
hơn
i
j
n
bằng 360◦ .
360◦
Do đó, ta có
> 60◦ ⇒ n < 6. Suy ra điều phải chứng minh.
n
Bài 4. Chứng minh rằng: Bốn hình trịn có đường kính là bốn cạnh của một tự giác lồi thì phủ
kín miền tứ giác ABCD.
Lời giải.
Lấy M là một điểm tùy ý của tứ giác lồi ABCD. Có hai khả năng xảy ra:
• Nếu M nằm trên biên của đa giác (tức M nằm trên một cạnh của tứ giác ABCD). Khi đó M
nằm trong hình trịn có đường kính là cạnh ấy. Trong trường hợp này kết luận của bài tốn hiển
nhiên đúng.




• Nếu M nằm bên trong tứ giác lồi ABCD. Khi đó ta có AM
B + BM

C + CM
D + DM
A = 360◦ .
Theo nguyên lí cực hạn, tồn tại





max{AM
B, BM
C, CM
D, DM
A} = BM
C.

Khi đó : BM
C ≥ 90◦ . (1)
Từ (1) suy ra M nằm trong ( hoặc cùng lắm là nằm trên) đường trịn đường kính BC. Vậy dĩ
nhiên M bị phủ bởi đường tròn này.
Như thế do M là điểm tùy ý của tứ giác ABCD, ta suy ra bốn hình trịn nói trên phủ kín tứ
giác lồi đã cho.

Bài 5. Có 3 nhóm học sinh, mỗi nhóm có n(n ≥ 2) học sinh. Biết rằng mỗi học sinh quen với
n + 1 học sinh ở hai nhóm cịn lại. Chứng minh rằng từ mỗi nhóm ta có thể chọn ra một học
sinh sao cho ba học sinh được chọn đôi một quen nhau.
Lời giải.
Trong các học sinh ta gọi A là học sinh mà có số người quen nhiều nhất với các học sinh trong
một nhóm khác.Giả sử A ở nhóm 1 và quen với k (k ≤ n) học sinh B1 , B2 , . . . , Bk ở nhóm 2.
Khi đó A sẽ quen với ít nhất một học sinh ở nhóm thứ 3. Ta giả sử A quen với C ở nhóm thứ

3. Vì C quen với khơng q k học sinh ở nhóm thứ nhất nên C sẽ quen với ít nhất n + 1 − k
học sinh ở nhóm thứ hai. Giả sử C quen với m học sinh B1 , B2 , . . . , Bm ở nhóm thứ 2 với
m ≥ n + 1 − k. Nếu hai tập {B1 , B2 , . . . , Bk } và B1 , B2 , . . . , Bm khơng có phần tử chung
thì ta có m + k ≤ n điều này vơ lí vì m + k ≥ n + 1 > n. Vậy tồn tại một học sinh B nằm ở hai
nhóm trên. Khi đó ba học sinh A, B, C ở ba nhóm và đơi một quen nhau. Bài tốn được chứng
minh.


4.. NGUN LÍ BẤT BIẾN

23

§4.

Ngun lí bất biến

Bất biến là đại lượng hay tính chất khơng thay đổi khi các trạng thái khác thay đổi. Bất biến có
nhiều ứng dụng trong toán học, nhất là trong các bài toán chứng minh tồn tại một trạng thái
hay tính chất nào đó qua một số lần thực hiện các thuật tốn.

I.

Ví dụ
Ví dụ 1. Lúc đầu ta ghi lên bảng cặp (1; 2). Từ lần thứ hai trở đi, nếu trên bảng có cặp


3a − b a + 3b
(a; b) thì ta được phép ghi cặp
;
. Chứng minh rằng không tồn tại cặp

2
2


3; 1 + 3 được ghi lên bảng?

Lời giải.
Đặt Sn = a2n + b2n .
Ở bước thứ n + 1 ta có:

Sn+1 =

3an − bn
2

2

+

Do đó, Sn là một đại lượng bất biến.


3
S1 = 5 =


nên không thể ghi được cặp
3; 1 + 3 .



an + 3bn
2

2

+ 1+



2

= a2n + b2n = Sn .

2

3 ,

Ví dụ 2. Trên bảng ghi hai số 1 và 2. Thực hiện cách ghi số theo quy tắc sau: Nếu trên
bảng có hai số a, b thì được ghi thêm số a + b + ab. Hỏi có thể ghi được số 2012 và 2013
lên bảng hay không?
Lời giải.
Ta ghi được các số 1, 2, 5, 11, 17, . . ..
Đặt c = a + b + ab ⇒ c + 1 = (a + 1)(b + 1) nên dãy số thu được công thêm 1 sẽ được các số có
dạng 2m 3n . Tuy nhiên 2013 và 2014 khơng có dạng đó nên ta khơng thể ghi được các số đó.
Ví dụ 3. Hình trịn được chia thành 2014 hình quạt. Mỗi hình quạt ta đặt một viên bị.
Ta thực hiện chuyển bị như sau: Mỗi lần lấy hai ô, mỗi ô một viên và chuyện qua ô bên
cạnh ngược chiều nhau. Hỏi với cách làm như vậy ta có thể chuyển tất cả các viên bi về
cũng một ô hay không?
Lời giải.
Tô màu các hình quạt bởi hai màu đen và trắng sao cho hai hình quạt kề nhau thì khác màu.

Khi đó mỗi lần chuyển thì số viên bi trong các ơ màu đen hoặc không đổi hoặc tăng 2 hoặc giảm
2. Hay nói cách khác số viên bi trong ơ đen ln cùng tính chẵn lẻ với số bi ban đầu ở trong ơ
đen. Mà lúc đầu có 1007 viên bi nằm trong các ô đen nên số bi ở các ô đen không thể là 0 và
2014 được. Do đó khơng thể thực hiên được u cầu của bài tốn.
Ví dụ 4. (IMO Shorlist C1, 2012) Một số số nguyên dương được viết trên một hàng.
Lựa chọn hai số kề nhau x, y sao cho x > y và x nằm bên trái của y và thay cặp (x; y) bởi
cặp (y + 1; x) hoặc (x − 1; x). Chứng minh rằng quá trình trên chỉ thực hiện được hữu hạn
lần.


24

CHƯƠNG 1. CÁC PHƯƠNG PHÁP GIẢI TOÁN

Lời giải.
Giả sử ta viết n số nguyên dương a1 , a2 , . . . , an và M = max{a1 , a2 , ..., an }. Đặt
S = a1 + 2a2 + . . . + nan .
Mỗi lần đổi ta đổi cặp (ai ; ai+1 ) với ai > ai+1 thành cặp (c; ai ) với c = ai+1 + 1 hoặc c = ai − 1.
Mỗi lần thay đổi như vậy thì tổng S thay đổi một lượng
d = ic + (i + 1)ai − (iai + (i + 1)ai+1 ) = (ai − ai+1 ) + i(c − ai+1 ) > 0
Do đó, sau mỗi lần thay thì tổng S tăng thêm ít nhất 1 đơn vị. Mà S ≤ (1 + 2 + . . . + n)M nên
đến một lúc nào đó q trình trên sẽ dừng lại. Vậy bài toán được chứng minh.

II.

Bài tập

Bài 1. Giả sử rằng n là một số lẻ. Đầu tiên ta viết các số từ 1 tới 2n trên một bảng đen. Sau
đó ta chọn ra hai số bất kì xố chúng và thay thế chúng bởi |a − b|. Chứng minh rằng số còn lại
cuối cùng là một số lẻ.

Lời giải.
Gọi S là tổng của tất cả các số trên bảng . lúc đầu ta có S = 1 + 2 + 3 + · · · + 2n = n(2n + 1) là
một số lẻ vì n là một số lẻ .
Ta cần tìm đại lượng bất biến .
Nhận thấy rằng sau mỗi lần thực hiện thuật toán như trong đầu bàI đã nói thì S sẽ bị mất đi
một đại lượng có giá trị bằng 2 · min(a; b). Vì thế tính chẵn lẻ của S được giữ nguyên sau mỗi
lần thực hiện thuật toán. Trong trường hợp của chúng ta thì S ln là một số lẻ và vì thế khi
trên bảng cịn lại một số thì số đó là số lẻ .
Bài 2. Một hình trịn được chia làm 6 cung. Viết các số 1, 0, 1, 0, 0, 0 nên các cung tròn (theo
chiều ngược chiều quay của kim đồng hồ ). Bạn có thể cộng hai số ở cạnh nhau với 1. Có thể
xảy ra trường hợp tới một lúc nào đó tất cả các số trên các cung trịn bằng nhau hay khơng ?
Lời giải.
Giả sử rằng sau một lúc nào đó trên hình trịn cịn lại các số a1 , a2 , a3 , a4 , a5 , a6 theo chiều kim
đồng hồ.
Với giả thuyết của bài toán, ta dễ dàng nhận thấy rằng đại lượng L = a1 − a2 + a3 − a4 + a5 − a6
ln khơng đổi (vì hiệu của 2 số ai và ai+1 luôn không đổi) và ban đầu L = 2. Vậy L luôn bằng
2. Trường hợp L = 0 không thể xảy ra.
Vậy dễ dàng nhận thấy các số trên các cung trịn khơng thể bằng nhau .
Bài 3. Xét dãy 1, 2, 3, 4, . . . , 2014. Mỗi lần ta xóa hai số a, b và viết thêm vào dãy số |a − b|. Sau
mỗi bước giảm một số. Hỏi số cịn lại cuối cùng có thể là số 10 hay không?
Bài 4. Cho các số 1, 2, 3, . . . , 2012 được xếp theo một thứ tự nào đó. Mỗi phép biến đổi cho
phép đổi thứ tự hai số kề nhau. Chứng minh rằng sau 2013 lần đổi chỗ khơng thể nhận được
hốn vị ban đầu.
Bài 5. Cho bộ ba số nguyên (a; b; c), ta xây dựng bộ mới (|a − b| ; |b − c| ; |c − a|). Chứng minh
rằng sau hữu hạn bước biến đổi ta được bộ gồm 3 số 0.
Bài 6. Giả sử n số thực n ≥ 4 được viết xung quanh một đường tròn. Nếu 4 số kề nhau a, b,
c, d thỏa (a − d)(b − c) < 0 thì ta đổi vị trí của b và c. Chứng minh rằng các phép biến đổi như
trên sẽ kết thúc sau hữu hạn bước.
Bài 7. Cho một bảng 1991 × 1992. Kí hiệu (m, n) là ơ vng nằm ở giao của hàng thứ m và cột
thứ n. Tô màu các ô vuông của bảng theo quy tắc sau: lần thứ nhất tô ba ô (r; s), (r + 1; s + 1),

(r + 2; s + 2) với 1 ≤ r ≤ 1989, 1 ≤ s ≤ 1990, từ lần thứ hai trở đi, mỗi lần tô đúng ba ô chưa
có màu nằm cạnh nhau trong cùng một hàng hoặc cùng một cột. Hỏi có thể tổ màu được tất cả
các ô hay không ?


4.. NGUYÊN LÍ BẤT BIẾN

25

Bài 8. Với bộ các số thực dương (a; b; c; d) ta thực hiện phép biến đổi T như sau
T

(a; b; c; d) → (ab; bc; cd; da).
Tìm bộ (a; b; c; d) ban đâu sao cho sau hữu hạn bước ta thu được bộ (a; b; c; d).
Lời giải.
k
Đặt P = abcd sau phép biến đổi thứ k ta thu được tích của bộ số P 2 Vì sau hữu hạn bước ta
thu được bộ ban đầu nên P = 1 hay abcd = 1 Ta có phép biến đổi
(a; b; c; d) → (ab; bc; cd; da) → ab2 c; bc2 d; cd2 a; da2 b → b2 c2 ; c2 d2 ; d2 a2 ; a2 b2 .
Đặt tk = max {ak ; bk ; ck ; dk } là giá trị lớn nhất của bộ tại bước thứ k.
k
Suy ra t2k = t2k = t22 . Vì sau hữu hạn bước ta thi được bộ ban đầu nên t2 = 1.
Suy ra
1 = ab.bc.cd.da ≤ t42 = 1 ⇒ ab = bc = cd = da = 1 ⇒ a = b = c = d = 1.
Rõ ràng với bộ bạn đầu là (1; 1; 1; 1) thì sau hữu hạn lần ta sẽ thu được bộ đó.


×